La ricerca ha trovato 79 risultati

da Pixel
03 giu 2005, 18:27
Forum: Teoria dei Numeri
Argomento: (frazioni)^3
Risposte: 14
Visite : 10315

ponendo q=$ \frac{m}{n} $ e $ a=m-k_1n $, $ b=m+k_1n $ $ c=n-k_2m $ e $ d=n+k_2m $ salta fuori qualcosa di carino, provate un pò.
da Pixel
03 giu 2005, 17:42
Forum: Teoria dei Numeri
Argomento: Equazioncina...
Risposte: 8
Visite : 7536

Mah proviamo il secondo: Osserviamo intento che l'equazione di partenza può essere scritta come: (x-y)(3x+3y+1)=y^2 , supponiamo ora che esista p primo tale che p|(x-y) e p|(3x+3y+1) . Abbiamo quindi: x=y+pk con k in N e sostituendo nella seconda relazione abbiamo 6y+1==0mod(p) . (1) Notiamo infine ...
da Pixel
27 mag 2005, 15:38
Forum: Matematica non elementare
Argomento: Simpatiche intersezioni.
Risposte: 17
Visite : 12600

Singollo, ti stanno solo prendendo in giro 8) :lol:
da Pixel
19 mag 2005, 16:56
Forum: Teoria dei Numeri
Argomento: Partizioni e verifiche di primalità
Risposte: 10
Visite : 8577

Beh mica era troppo piccola come svista :evil: 8) Comunque provo la prima implicazione: Supponiamo dunque n numero primo, supponiamo per assurdo che esista (x_1,x_2,x_3,x_4) di interi positivi tali che n=x_1+x_2+x_3+x_4 e tale per cui vi sia una permutazione {p,q,r,s} di {1,2,3,4} tale che x_px_q=x_...
da Pixel
17 mag 2005, 18:19
Forum: Geometria
Argomento: Una relazione notevole (..secondo me)
Risposte: 7
Visite : 6782

L'angolo PAB..etc va considerato con il vertice in A?
da Pixel
15 mag 2005, 15:11
Forum: Teoria dei Numeri
Argomento: Partizioni e verifiche di primalità
Risposte: 10
Visite : 8577

Ciao! Un'implicazione viene abbastanza agevolmente...se n è primo allora... Per quanto riguarda l'altra sei sicuro di non aver omesso ipotesi aggiuntive? Cioè prendendo n=20 e come quadrupla (2,4,8,6) mi pare che funzioni, ma 20 è ben lungi dall'essere primo. Boh..magari ho frainteso! P.S: Qual è il...
da Pixel
14 mag 2005, 17:33
Forum: Teoria dei Numeri
Argomento: Successioni e coprimalità II: the revenge
Risposte: 2
Visite : 3745

Equivalentemente si poteva dimostrare per induzione che per ogni m:

$ (a_{m},a_{n})=(a_{n},k) $ fissato n
quindi dimostrare che per ogni n:
$ (a_{n},k)=1 $ e avere la tesi.
da Pixel
14 mag 2005, 13:12
Forum: Teoria dei Numeri
Argomento: Partizioni e verifiche di primalità
Risposte: 10
Visite : 8577

Ciao Salvo!

Lo so che sono OT, ma mi chiedevo se potessi descrivermi l'oggetto dello studio della teoria geometrica dei numeri, insomma qual è il genere d'approccio e qualche risultato notevole.

Grazie

P.S: forse era meglio metterla nel glossario, boh!.. parola ai mods

Ancora ciao
da Pixel
10 mag 2005, 16:16
Forum: Olimpiadi della matematica
Argomento: Risultati dei Bresciani??
Risposte: 30
Visite : 23748

D'oh sì sì certo!

Altrimenti cade l'ipotesi che AB, BC e BH siano lati di un triangolo rettangolo!

Sorry :oops:
da Pixel
10 mag 2005, 11:51
Forum: Olimpiadi della matematica
Argomento: Risultati dei Bresciani??
Risposte: 30
Visite : 23748

Ciao ricapitolando le soluzioni del problema 1 sono:

a) $ \frac{2}{1+\sqrt(5)} $ se $ AB<BC $
b) $ \frac{1+\sqrt(5)}{2} $ se $ BC<AB $
c) $ 1 $ se $ AB=BC $

No?
da Pixel
29 apr 2005, 14:47
Forum: Matematica non elementare
Argomento: topologia
Risposte: 4
Visite : 5156

Scusami Marco ma non capisco il senso di quello che hai scritto.
Cito testualmente:
Dimostrare che il grafico di una funzione f: I-->R è compatto se e solo se f è continua.
(I è l'intervallo [0,1])

Ciao
da Pixel
29 apr 2005, 13:52
Forum: Matematica non elementare
Argomento: topologia
Risposte: 4
Visite : 5156

topologia

Dimostrare che un'applicazione f: I-->R è continua se e solo se il suo grafico è compatto.

Ciao
da Pixel
16 apr 2005, 18:40
Forum: Teoria dei Numeri
Argomento: Cubi e diofantee
Risposte: 31
Visite : 22355

o se p è appunto razionale :D
da Pixel
16 apr 2005, 18:32
Forum: Teoria dei Numeri
Argomento: Cubi e diofantee
Risposte: 31
Visite : 22355

No Spider era rivolto al risolutore!

Infatti a,b,c sono in generale numeri razionali e DarkskyO sembra infischiarsene di brutto trattandoli come interi...credo sia qui l'errore della dimostrazione
da Pixel
16 apr 2005, 17:28
Forum: Teoria dei Numeri
Argomento: Cubi e diofantee
Risposte: 31
Visite : 22355

Occhio che lavori con i razionali!!! 8)